You are on page 1of 3

\documentclass{article}

\usepackage{amsmath,amssymb,amsthm,latexsym,paralist}
\theoremstyle{definition}
\newtheorem{problem}{Problem}
\newtheorem*{solution}{Solution}
\newtheorem*{resources}{Resources}
\newcommand{\problemset}[1]{\begin{center}\textbf{Problem Set #1}\end{center}}
\newcommand{\N}{\mathbf{N}}
\newcommand{\R}{\mathbf{R}}
\newcommand{\Z}{\mathbf{Z}}

\begin{document}
\vspace*{-15mm}
\begin{center}
{\large
TEST
\end{center}
\problemset{1}
\honor
\smallskip
\begin{problem} Section 1.1, Exercise 6 on page 13.
\end{problem}
\begin{solution}
\hangindent=1.7cm
\textbf{a.)} T \\
\textbf{b.)} T \\
\textbf{c.)} F \\
\textbf{d.)} F \\
\textbf{e.)} F
\end{solution}
\begin{problem} Section 1.1, Exercise 16 on page 14.
\end{problem}
\begin{solution}
\hangindent 1.7cm
\textbf{a.)} T \\
\textbf{b.)} F \\
\textbf{c.)} T \\
\textbf{d.)} F
\end{solution}
\begin{problem} Section 1.1, Exercise 22 a) -- e) on page 14.
\end{problem}
\begin{solution} \hangindent 1.7cm
\textbf{a.)} If you get promoted, then you must have washed the boss's car.
\\
\textbf{b.)} If the winds are from the south, then here is a spring thaw. \\
\textbf{c.)} If you bought your computer less than a year ago, then the warr
anty is still good. \\
\textbf{d.)} If Willy cheats, then he gets caught. \\
\textbf{e.)} If you can access the website, then you have paid a subscriptio
n fee.
\end{solution}
\begin{problem} Section 1.1, Exercise 40 on page 16.
\end{problem}
\begin{solution} \hangindent 1.7cm
\textbf{a.)} going from left to right... In (p $\lor$ $\neg$q) you can eithe
r choose to go true or false values. If we choose true, so p,
then we have bump into (r $\lor$ $\neg$p). These are connected with $\land$. So
now we have p $\land$ r to keep the proposition true. Next we bump into (q $\lo
r$ $\neg$r). Again, this proposition is connected with $\land$, so to keep the p
roposition true, we need q. So we have p , q , r evaluate to true. But if we sta
rt off with $\neg$q then we need to choose $\neg$r and therefor we need to also
choose $\neg$p.
\end{solution}
\begin{problem} Section 1.1, Exercise 42 on page 16. \textsl{Explain.}
\end{problem}
\begin{solution} \hangindent 1.7cm
\textbf{a.)} x = 2 \\
x + 2 = 3 is T \\
\textbf{b.)} x = 1 \\
F and F so x doesn't get assigned \\
\textbf{c.)} x = 2 \\
T and T so x -> 2 \\
\textbf{d.)} x = 1 \\
T and T, but exclusive, so F \\
\textbf{e.)} x = 2 \\
T
\end{solution}
\begin{problem} Section 1.2, Exercise 6 on page 22.
\end{problem}
\begin{solution} \hangindent 1.7cm
\textbf{a.)} $u \rightarrow [[b_{32} \land (g_1 \lor g_2) \land (r_1 \lor r_
2) \land (h_{16} \lor h_{32})] \lor (b_{64} \land g_2 \land r_2 \land h_{32})]$
\end{solution}
\begin{problem} Section 1.2, Exercise 22 on page 23. \textsl{Explain.}
\end{problem}
\begin{solution} \hangindent 1.7cm
\textbf{a.)} There is no way to conclude who is whom. They could both be Kni
ghts telling the truth (saying that they are both knights) or they coul
d both be knaves lying (therefor saying they are both knights). There is als
o the chance for them to be one knave and one knight...
\end{solution}
\begin{problem} Section 1.3, Exercise 10 b) and c), page 35.
\end{problem}
\begin{solution} \hangindent 1.7cm
\textbf{b.)}
\begin{displaymath} \begin{array}{|c c c|c|c|c|c|c|} % we need 5 coloumns
p & q & r & p \rightarrow q & q \rightarrow r & (p \rightarrow q
) \land (q \rightarrow r) & (p \rightarrow r) &
[(p \rightarrow q) \land (q \rightarrow r)] \rightarrow [(p \rightarrow r)] \\
\hline
T & F & F & T & T & T & T & T \\
F & F & F & T & T & T & T & T \\
T & T & F & F & T & F & F & T \\
F & T & F & F & T & F & F & T \\
F & T & T & T & F & F & F & T \\
T & T & T & T & T & T & T & T \\
F & F & T & T & F & F & T & T \\
T & F & T & F & T & F & T & T \\
\end{array} \end{displaymath} \\
\textbf{c.)}
\begin{displaymath} \begin{array}{|cc|c|c|c|}
P & Q & P \to Q & P \land (P \to Q) & [P \land (P \to Q)] \to Q \\
\hline
T & T & T & T & T \\
T & F & F & F & T \\
F & F & T & F & T \\
F & T & T & F & T \\
\end{array} \end{displaymath}

\end{solution}
\noindent
Solve the following two problems by developing a series of logical equivalences,
as shown in class and also in Examples~7 and~8 on page 30.
\begin{problem} Show that $(p\rightarrow q) \lor (p\rightarrow r)$ and
$p\rightarrow (q\lor r)$ are logically equivalent.
\end{problem}
\begin{solution} \hangindent 1.7cm
\textbf{$(p \rightarrow q) \lor (p \rightarrow r) \equiv p \rightarrow (
q \lor r)$} \\
\line(1,0){130} \\
$\equiv (\neg p \lor q) \lor (\neg p \lor r)$ \\
$\equiv \neg p \lor q \lor \neg p \lor r$ \\
$\equiv \neg p \lor q \lor r$ \\
$\equiv \neg p \lor (q \lor r)$ \\
\textit{make $(q \lor r) = x$} \\
$\equiv p \to x$ \\
$\equiv p \to (q \lor r)$ \\
\end{solution}
\begin{problem} Show that $(p \lor q)\land (\neg p\lor r)\rightarrow (q\lor r)$
is a tautology.
\end{problem}
\begin{solution} \hangindent 1.7cm
$(p \lor q)\land (\neg p\lor r)\rightarrow (q\lor r) \equiv T$ \\
\line(1,0){150} \\
$\equiv \neg[(p \lor q) \land (\neg p \lor r)] \lor (q \lor r)$ \\
$\equiv \neg(p \lor q) \lor \neg(\neg p \lor r) \lor (q \lor r)$ \\
$\equiv (\neg p \land \neg q) \lor (p \land \neg r) \lor (q \lor r)$ \\
$\equiv q \lor (\neg p \land \neg q) \lor r \lor (p \land \neg r)$ \\
$\equiv (q \lor \neg p) \land (q \lor \neg q) \lor (r \lor p) \land (r \lor
\neg r)$ \\
$\equiv T \land (q \lor \neg p) \lor T \land (r \lor p)$ \\
$\equiv q \lor \neg p \lor r \lor p$ \\
$\equiv q \lor T \lor r$ \\
$\equiv T$ \\
\end{solution}
\goodbreak
\checklist
\end{document}

You might also like